subject
Mathematics, 08.02.2021 16:10 derpoofoofderp

Let y1, y2,... , yn denote a random sample of size n from a population with a uniform distribution on the interval (0,theta). let y(n) = max{y1,y2,...,yn} and u = (1/theta)y(n). (a) show that u has distribution function fu(u) = { (b) because the distribution of u does not depend on theta, u is a pivot. find a 95% lower confidence hound for theta.

ansver
Answers: 1

Another question on Mathematics

question
Mathematics, 21.06.2019 16:00
Question part points submissions used suppose that 100 lottery tickets are given out in sequence to the first 100 guests to arrive at a party. of these 100 tickets, only 12 are winning tickets. the generalized pigeonhole principle guarantees that there must be a streak of at least l losing tickets in a row. find l.
Answers: 2
question
Mathematics, 21.06.2019 16:30
What is the binomial expansion of (2x-3)^5
Answers: 1
question
Mathematics, 21.06.2019 18:00
What is the upper quartile in the box plot?
Answers: 1
question
Mathematics, 21.06.2019 18:30
Complex numbers multiply √-4 * √-25 and show all intermediate steps. alternative notation is sqrt(-4) * sqrt(-25).
Answers: 1
You know the right answer?
Let y1, y2,... , yn denote a random sample of size n from a population with a uniform distribution o...
Questions
question
Computers and Technology, 07.08.2019 22:10
question
Computers and Technology, 07.08.2019 22:10